Using Laplace Transform to Solve Non-Zero Initial Condition PDEs

Click For Summary
The discussion revolves around solving a system of ordinary differential equations using the Laplace Transform, specifically addressing non-zero initial conditions. The initial confusion stems from incorrectly applying the Laplace Transform, particularly in the treatment of derivatives and initial values. The correct approach involves using the formula L(x') = sX(s) - x(0) and similarly for y', while also transforming the non-homogeneous term (t^2 - 2t + 1)e^{-(t-1)}. Participants suggest leaving the initial conditions as variables in the equations until the final solution, where they can be substituted to satisfy the given conditions. This clarification helps resolve previous misunderstandings and leads to a more coherent solution process.
mike1111
Messages
8
Reaction score
0

Homework Statement


Help, I don't know how to do the following question:

Using Laplace to solve
x' -y =1
2x' +x +y' = (t2-2t+1)e-(t-1)

Homework Equations


x(1)=0
y(3)=0


The Attempt at a Solution


The problem I'm having is the initial conditions aren't at zero, and I'm not sure how to approach the question

so far I have:
X(s) -x(0) -Y(s) = 1/s
3X(s) -2x(0) +Y(s)-y(0)= F{(t2-2t+1)e-(t-1)}
 
Physics news on Phys.org
This isn't a pde. It is a system of ordinary DE's. And you have mistakes in your transforms.

L(x') is not X(s) - x(0) it is sX(s) - x(0), and ditto for y'. And, of course, you need to transform (t2-2t+1)e-(t-1).

Just call x(0) = a and y(0) = b and leave them in there. Once you get the equations right and solve for X(s) and Y(s), you can take the inverse transforms. Your answers will have a and b in them. Finally, plug in your given initial conditions and choose a and b to make them work.
 
Thanks a lot LCKurtz, I didn't see that mistake. and the solution makes more sense now. I was getting weird answer from other questions too and could work out why.
 
Question: A clock's minute hand has length 4 and its hour hand has length 3. What is the distance between the tips at the moment when it is increasing most rapidly?(Putnam Exam Question) Answer: Making assumption that both the hands moves at constant angular velocities, the answer is ## \sqrt{7} .## But don't you think this assumption is somewhat doubtful and wrong?

Similar threads

  • · Replies 7 ·
Replies
7
Views
2K
  • · Replies 10 ·
Replies
10
Views
2K
Replies
2
Views
2K
Replies
9
Views
2K
  • · Replies 8 ·
Replies
8
Views
2K
Replies
2
Views
2K
Replies
2
Views
2K
  • · Replies 3 ·
Replies
3
Views
3K
  • · Replies 1 ·
Replies
1
Views
1K
Replies
4
Views
2K